Apply the mean value theorem for integrals

Click For Summary
The discussion revolves around applying the Mean Value Theorem for integrals, specifically addressing the confusion regarding the steps involved in solving the problem. Key points include the correct calculation of the antiderivative of the function 4 - x, which is 4x - x^2/2, and the necessity of multiplying 4 by (b - a) to evaluate the integral. The distributive law is highlighted as a crucial mathematical principle for simplifying expressions during integration. Additionally, the properties of integrals are discussed, emphasizing how they can be applied to break down the problem into manageable parts. Overall, the conversation clarifies the steps needed to correctly apply the theorem and understand the underlying mathematical concepts.
bobsmith76
Messages
336
Reaction score
0

Homework Statement



Screenshot2012-02-01at22839PM.png




The Attempt at a Solution



My book is not explaining very well the steps at solving these problems. There is a step that I'm missing

step 1. find 1/(b-a) easy
step 2. find the antiderivative of 4 - x, easy, x^2/2
step 3. plug in what the result of the antiderivative is for b and a, subtract b from a
step 4. this is the step I'm missing. the books shows that I'm supposed to multiply 4 by b - a. why? what is this? I need to know what mathematical terminology this is so that i can do these problems in the future
step 5. subtract step 4 from step 3. why? what mathematical terminology is this.
step 6. multiply step 1 by step 5, I understand that.

I'm not yet able to read the notation and the book is not helping.
 
Physics news on Phys.org
bobsmith76 said:

Homework Statement



Screenshot2012-02-01at22839PM.png




The Attempt at a Solution



My book is not explaining very well the steps at solving these problems. There is a step that I'm missing

step 1. find 1/(b-a) easy
step 2. find the antiderivative of 4 - x, easy, x^2/2
Well, 4x- x^2/2

step 3. plug in what the result of the antiderivative is for b and a, subtract b from a
step 4. this is the step I'm missing. the books shows that I'm supposed to multiply 4 by b - a. why? what is this? I need to know what mathematical terminology this is so that i can do these problems in the future
The anti-derivative of 4, with respect to x, is 4x. That is evaluated between the limits of integration, b and a. You can do it as either 4b- 4a or 4(b- a). They are equal by the "distributive law".

step 5. subtract step 4 from step 3.
No, they are not subtracting step 4 from step 3. They are simply completing the calculation indicated in step 4. There are, however, different ways of doing that step. You could have calculated that 4(3)= 12, and 3^2/2= 9/2 and subtract: 12- 9/2= 24/2- 9/2= 15/2.
Then multiply by the leading 1/3: (1/3)(15/2)= 5/2.

Or, using the distributive law, go ahead and multiply by the 1/3 first: (1/3)(4(3)- 3^2/2)= (1/3)(4(3))- (1/3)(3^2)/2. That 1/3 will cancel a 3 in eacy term: it is equal to 4- 3/2= 8/2- 3/2= 5/2 again.

why? what mathematical terminology is this.
step 6. multiply step 1 by step 5, I understand that.

I'm not yet able to read the notation and the book is not helping.
The distributive law: a(b+ c)= ab+ ac.
 
Ok, thanks, I incorrectly calculated the antiderivative of 4 - x as (x^2)/2, it's (4x - x^2)/2.

Thanks for your help
 
All that's happened here is an integral
Here are some properties of integrals that you should know that will also help you understand your problem

Let a be a constant, f(x) be some function of x, and F(x) be it's anti-derivative

1. \int (a \pm f(x))dx = \int a dx \pm \int f(x) dx

2. \int a f(x) dx = a \int f(x) dx

3. \int 1 dx = x

4. \int_a ^b f(x) dx = F(b) - F(a)Now I shall show you which of these properties we shall use in our problem;

\frac{1}{b-a} \int_a ^b f(x) fx = \frac{1}{b-a} \int_a ^b (4-x)dx

I shall use property 1. now

\frac{1}{b-a} \int_a ^b (4-x)dx = \frac{1}{b-a}( \int_a ^b 4 dx - \int_a ^b x dx[/itex]

I shall now use property 2. on the first term

\frac{1}{b-a}( \int_a ^b 4 dx - \int_a ^b x dx = \frac{1}{b-a} ( 4 \int _a ^b 1 dx - \int_a ^b x dx)

Due to property 3, the anti-derivative of 1 is x (or in the terms of property 4. if f(x)=1 then F(x) = x)
And using property 4 to evaluate the definite integrals

\frac{1}{b-a} ( 4 \int _a ^b 1 dx - \int_a ^b x dx) = \frac{1}{b-a} ( 4(b-a) -( \frac{b^2}{2} -\frac{a^2}{2} ))

Substituting in our values for a and b we get

\frac{1}{b-a} ( 4(b-a) -( \frac{b^2}{2} -\frac{a^2}{2} )) = \frac{1}{3} ( 4(3) -\frac{3^2}{2}) = 4 - \frac{3}{2} = \frac{5}{2}

Has this helped at all?
 
Question: A clock's minute hand has length 4 and its hour hand has length 3. What is the distance between the tips at the moment when it is increasing most rapidly?(Putnam Exam Question) Answer: Making assumption that both the hands moves at constant angular velocities, the answer is ## \sqrt{7} .## But don't you think this assumption is somewhat doubtful and wrong?

Similar threads

Replies
3
Views
2K
Replies
7
Views
2K
  • · Replies 17 ·
Replies
17
Views
2K
  • · Replies 3 ·
Replies
3
Views
1K
Replies
5
Views
919
  • · Replies 7 ·
Replies
7
Views
2K
  • · Replies 3 ·
Replies
3
Views
2K
  • · Replies 14 ·
Replies
14
Views
3K
  • · Replies 4 ·
Replies
4
Views
1K
  • · Replies 3 ·
Replies
3
Views
1K